Kaplan Flashcards
A 54-year-old man presents to the physician’s office complaining of a sore on his foot that “just won’t heal.”The cut has been present for the last month and is at the tip of his toe. There is little pain associated with the wound. He has smoked 2 packs of cigarettes per day for the last 30 years and resists quitting, although he has been warned about it multiple times due to his claudication. His vital signs in the office are: temperature 98.6°F (36°C), blood pressure 112/76 mm Hg, pulse 87/min, respiratory rate 20/min. Physical examination reveals bilateral expiratory wheezes in all fields. Examination of the foot reveals a dirty-appearing wound on the tip of the big toe; the wound measures 5 mm in diameter with a pale base and absent granulation tissue. No erythema, pus, or signs of cellulitis are noted. Pulses cannot be felt bilaterally but monophasic flow can be heard with mini-Doppler. Which of the following is the best next step in the management of this patient?
A. Bilateral arteriograms of the lower extremities
B. Above-the-knee amputation
C. Surgical revascularization
D. Support stockings
E. Wide local excision
La respuesta es la A
La herida es una ulcera por insuficiencia arterial
An 11-year-old Latino girl was found unconscious in the desert by Border Patrol officers and was immediately transported to the closest medical facility. Upon arrival, her skin looks pale, dry, and hot; she is tachypneic and has weak pulses. Physical exam reveals a systolic ejection murmur at the mid-sternal border. A fixed wide splitting of S2 and a low-pitched diastolic rumble over the lower right sternal border are also present. Chest x-ray shows a large heart and increased pulmonary vascularity. Echocardiogram shows right ventricular hypertrophy. This patient most likely has which of the following heart anomalies?
A. Atrial septal defect (ASD)
B. Ventricular septal defect (VSD)
C. Patent ductus arteriosus (PDA)
D. Aortic stenosis
E. Tetralogy of Fallot
La respuesta es la A: defecto en el septum auricular:
- Soplo sistólico que se oye mejor en el borde mid sternal, S2 doblado.
- La rx de torac puede salir normal o mostrar cardiomegalia.
- El ecocardiograma puede mostrar hipertrofia ventricular
A 71-year-old man arrives in the emergency department complaining of nausea, vomiting,
and visual disturbances. He claims his vision is distorted and he has been seeing “yellowish” hues. He has noticed these symptoms progressing over the last 6 to 8 hours. He denies chest pain, although he has felt occasional palpitations. He has felt lightheaded but has not had frank syncope. His past medical history is significant for congestive heart failure, coronary artery disease, and hypertension. His medication regimen includes atorvastatin, metoprolol, digoxin, and furosemide. He was recently started on a 7-day course of erythromycin for a presumed sinusitis. Physical examination demonstrates a blood pressure of 90/60 mm Hg and a pulse of 40/min. Head and neck examination demonstrates an elevated jugular venous pressure. He has bibasilar crackles and a third heart sound. His abdominal examination is benign. Extremity examination demonstrates weak pulses and pedal edema. Laboratory studies are obtained and shown below:
Sodium 138 mEq/L
Potassium 2.9 mEq/L
Chloride 105 mEq/L
Bicarbonate 22 mEq/L
Urea nitrogen 19 mg/dL
Creatinine 1.5 mg/dL
Which of the following would be most detrimental to this patient at this time?
A. Hypermagnesemia
B. Hyperphosphatemia
C. Hypocalcemia
D. Hypomagnesemia
E. Hypophosphatemia
La respuesta es D porque el px esta intoxicado por digoxina. El magnesio es un cofactor de la bomba NaK ATPasa, y en su ausencia puede potenciar la acción tóxica de la digoxina
A 25-year-old woman arrives in urgent care clinic because of shortness of breath. The dyspnea has been progressive and has been especially bad over the past week. She has noticed that it is nearly impossible to go from her recliner to the bathroom, whereas just 1 month before she was able to run 5 miles a day. She has also noticed both her ankles swelling. Her past medical history is unremarkable, and she takes no medications. She has not had any recent travel. Her family history is unremarkable also, with no significant cancer or heart disease. Further questioning reveals an upper respiratory infection approximately 2 weeks before. On physical examination, she is afebrile, her blood pressure is 100/60 mm Hg and her pulse is 115/min. She has jugular venous distension up to the level of her jaw. Diffuse crackles are auscultated. She has no murmurs, rubs, or gallops. Her abdomen is slightly distended, with a positive fluid wave. She has bilateral lower extremity 2+ pitting edema. An echocardiogram is performed that demonstrates an ejection fraction of 20% with normal wall thickness. She has an elevated serum creatine kinase. A chest radiograph demonstrates pulmonary edema. Which of the following will give the most definitive diagnosis?
A. Coronary catheterization
B. Myocardial biopsy
C. Pericardiocentesis
D. Pulmonary angiography
E. Pulmonary artery catheterization
La respuesta es la B: La px tiene miocarditis. Tiene signos y sintomas consistentes con falla izquierda severa. Tiene edema pulmonar y signos de presi{on derecha elevada, como la ingurgitaci{on yugular, ascitis, y edema de miembros inferiores.
El gold standard para la miocarditis es la biopsia del miocardio.
A 79-year-old woman is hospitalized for community acquired pneumonia. She was in her usual state of health when she developed high spiking fevers, a productive cough, and malaise. She was begun on an intravenous fluoroquinolone, and she stopped having fevers and began to feel better. Her past medical history is notable for hypertension, for which she takes hydrochlorothiazide and lisinopril as an outpatient. During the second night of her hospitalization, she develops a “funny feeling” in her chest. Her pulse demonstrates a rate of 130/min, and her blood pressure is 125/75 mm Hg. She denies feeling dizzy or faint, but the feeling persists. A bedside electrocardiogram demonstrates a narrow-complex tachycardia confirming a heart rate of approximately 130/min. Her lung examination demonstrates some residual rhonchi over the right middle lobe, which is the site of her pneumonia, but is otherwise unremarkable. No cardiac murmur is appreciated. Her abdominal and extremity examination is unremarkable. Which of the following should be administered to this patient?
A. 1 mg of intravenous epinephrine
B. External cardioversion
C. Intravenous adenosine
D. Intravenous atropine
E. Oral metoprolol
La respuesta es la C: el px tiene una taquicardia supraventricular, esto por el qrs angosto. Debido a la historia del px de hta lo mas probable es que tenga una FA. La adenosina bloquea el nodo av por un mini rato.
A 59-year-old man with a history of type 2 diabetes mellitus and hypertension comes to the
physician for a routine physical examination. He has tried to exercise three times a week but has not been successful in doing so. He smokes cigarettes 2-3 times per week and admits to eating an unhealthy diet. Current medications include hydrochlorothiazide and metformin. His temperature is 37.1°C (98.8°F), pulse is 75/min, and blood pressure is 148/96 mm Hg. Body mass index (BMI) is 30.4 kg/m2. Physical examination shows no abnormalities. Which of the following is the next best step in management?
A. Add lisinopril to his regimen
B. Add metoprolol to his regimen
C. Recommend smoking cessation and weight loss
D. Recommend the DASH (dietary approaches to stop hypertension) diet
E. Replace hydrochlorothiazide with hydralazine
La respuesta es la A: el px con historia de hta y con hta en el examen físico, según la AHA requieren manejo farmacológico. En px con dm2 y hta se le debe dar un ieca como el lisinopril ya que ayuda con la hta y es nefroprotector. Los iecas inhiben la acci{on de la angiotensina II en la arteriola eferente, lo que disminuye la presión glomerular y reduce la proteinura.
A 26-year-old man is brought to the emergency department after a syncopal episode that
occurred during exercise. He immediately regained consciousness, and there was no
seizure activity noted. His blood pressure is 130/70 mm Hg. Physical examination shows a
brisk upstroke in the carotid pulse. There is a systolic ejection murmur with a systolic thrill at the left lower sternal border. The intensity of the murmur increases with standing. Which of the following is the most appropriate next step in the evaluation of this patient?
A. Chest radiograph
B. Echocardiography
C. Pulmonary function tests
D. Stress test
E. Ultrasound of the abdominal aorta
La respuesta es la B: la px tuvo un episodio de síncope, ademas tiene un soplo sistólico que aumenta con la bipedestación, por lo cual lo mas probable es que tenga cardiomiopatía hipertrófica. Se le debe hacer un ecocardiograma para confirmar el diagnostico y evaluar el grosor de la pared del ventrículo derecho.
A 46-year-old man with a history of hypertension and hypercholesterolemia comes to the physician for a routine follow-up. The patient’s job involves a lot of traveling, and he admits to occasionally forgetting to take his medications with him when he travels. He reports several episodes of chest pain in the past few months. The pain is sharp in nature, mainly over his lower chest and epigastrium, and tends to come on with walking. He believes these episodes are due to indigestion and has been taking antacids. There is a family history of heart disease, and his father died of a heart attack at age 48. His pulse is 86/min and blood pressure is 150/80 mm Hg. His lungs are clear to auscultation. Cardiac auscultation shows normal rate and rhythm without murmurs, rubs, or gallops. There is no pedal edema. He is sent for an exercise stress test. Five minutes into the test, he develops ST depression of 3 mm in leads V1 V5. The length of the ST-segment depression is greater than 0.12 seconds in duration, and the stress test is stopped. Which of the following is the most appropriate next step in management?
A. Coronary angiography
B. Dobutamine stress test
C. Echocardiogram
D. Holter monitor
E. Repeat stress test in 2 weeks
La respuesta es la A: Los px con prueba de estrés positiva requieren una angiografía coronaria con el fin de buscar la coronaria tapada y definir la revascularizaci{on
A 62-year-old man is brought to the emergency department because of crushing chest pain. His blood pressure is 106/62 mm Hg and his pulse is 100/min. Physical examination shows diaphoresis. Cardiopulmonary examination shows positive S3 and S4, as well as crackles heard bilaterally over the lower one-third of the lung fields that clear with coughing. Peripheral pulses are palpable and symmetric. There is no peripheral edema. An ECG shows Q waves without ST elevation. Troponin and CK-MB are significantly elevated. Which of the following is the most appropriate next step in management?
A. Administer 325 mg of chewable aspirin
B. Begin therapy with streptokinase
C. Insert a pulmonary artery catheter
D. Perform intra-aortic balloon counterpulsation
E. Perform technetium-99m pyrophosphate scintigraphy
La respuesta es la A: Este ox probablemente tiene un SCASEST, y a ellos se le debe dar aspirina
A 64-year-old woman comes to her physician for a follow-up of hypertension and congestive heart failure. She states that she uses 2 pillows to sleep at night and has occasional shortness of breath that wakes her from sleep. She is currently taking furosemide and carvedilol. She was recently hospitalized for pulmonary edema. An echocardiogram at that time showed an ejection fraction of 35–40% with a globally hypokinetic left ventricle. Her blood pressure is 150/80 mm Hg and pulse is 80/min. Physical examination shows clear lungs and no extra heart sounds. The point of maximal impulse is laterally displaced. There is no peripheral edema. Which of the following is the most appropriate next step in management?
A. Add an ACE inhibitor to her regimen
B. Add amlodipine to her regimen
C. Add hydralazine to her regimen
D. Decrease the dose of her beta-blocker
E. Increase the dose of her loop diuretic
La respuesta es la A: la px tiene HTA y falla cardiaca. Los IECA ayudan con tasa de sobrevivencia en falla cardiaca en px con FE menor a 40%. Ademas ayuda con la HTA
A 45-year-old man presents to the emergency room with a chief complaint of bright-red blood in his stools every 4 hours since this morning. He has never had this problem before. He has a past medical history significant for multiple rehabilitations due to alcoholism. During the interview, the man excuses himself to the bathroom, where he has another bright-red bowel movement. His vital signs are: temperature 98.6°F (36.0°C), blood pressure 110/70 mm Hg, pulse 95/min, respiratory rate 14/min. Examination reveals a tall, thin man in moderate distress. His heart, lung, and abdomen exams are benign. Which of the following is the best next step in the management of this patient?
A. Place a nasogastric tube
B. Upper gastrointestinal endoscopy
C. Anoscopy
D. Sigmoidoscopy
E. Immediate surgical consult
La correcta es la A: en caso de que cualquier px con hematoquezia, el origen es del TGI. El primer paso es mirar signos y si el px esta estable, si sio se debe pasar sonda nasogastrica y aspirar el contenido gástrico.
A hysterical mother comes to the emergency room because her 5-year-old son swallowed a small, buttonshaped battery 5 hours ago. She threatens to sue everybody in the hospital if her son is not evaluated immediately. The child does not look distressed and has no difficulty breathing, but he complains of a weird sensation in his chest. A chest x-ray shows the battery lodged in the lower esophagus. Which of the following is the most appropriate next step in management?
A. Expectant management for 24 hours to see if the battery passes to the stomach on its own
B. Immediate endoscopy and removal of the battery
C. Surgical removal of the battery
D. CT scan of the abdomen
E. Catheter guided by fluoroscopy to remove the batter
La respuesta es B: las baterías pequeñas deben ser sacadas ya del esofago para evitar complicaciones.